Holomorphe Fkt auf Einheitskreisscheibe

Neue Frage »

Dunkit Auf diesen Beitrag antworten »
Holomorphe Fkt auf Einheitskreisscheibe
Hi!
Ich habe ne Aufgabe und ne Vermutung dazu, aber bin mir nicht ganz sicher...

Also es ist holomorph (wobei D die Einheitskreisscheibe) und .
Frage: Ist beschränkt?

Ich meine nein, denn:
betrachte .
Da in 0 ja eine wesentliche Singularität hat, gilt für in 1 dasselbe, oder?!
Außerdem ist beschränkt, wie in der Vor. angegeben.
Da eine wesentliche Sing. in 1 hat ist das Bild einer bel. kleinen Punktierten Umgebung um 1 also dicht in also sicher nicht beschränkt.

Sehe ich da jetzt irgendwas falsch? Ich bin mir da nicht so sicher, gerade was die Voraussetzung angeht, ob die auf meine Funktion zutrifft...
Sly Auf diesen Beitrag antworten »

Ich würde sagen, ja:

Jede Einschränkung ist beschränkt. Wenn f unbedschränkt wäre, müsste es schon eine Folge in D geben, sodass ...
Kann das hier sein?
Dunkit Auf diesen Beitrag antworten »

Naja Moment für ist sie doch gerade nicht notwendigerweise beschränkt?!
system-agent Auf diesen Beitrag antworten »

Deine Funktion ist in nicht einmal definiert, also insbesondere keine Abbildung von irgendwohin...
Dunkit Auf diesen Beitrag antworten »

ja, also irgendwie funktioniert das alles nicht so wirklich. Aber mir fällt weder ein Beweis, noch ein funktionierendes Gegenbeispiel ein... Ich weiß ja noch nichtmal, nach was von beidem ich suchen soll Augenzwinkern
Sly Auf diesen Beitrag antworten »

Zitat:
Original von system-agent
Deine Funktion ist in nicht einmal definiert, also insbesondere keine Abbildung von irgendwohin...

D ist ja auch die Einheitskreisscheibe, die Abbildung ist also sehr wohl definiert

Zitat:
ja, also irgendwie funktioniert das alles nicht so wirklich. Aber mir fällt weder ein Beweis, noch ein funktionierendes Gegenbeispiel ein... Ich weiß ja noch nichtmal, nach was von beidem ich suchen soll


Jetz lies dir doch noch mal meinen Post durch...da steht doch schon einiges drin Augenzwinkern
 
 
Dunkit Auf diesen Beitrag antworten »

Naja ein bisschen sorge mache ich mir schon um die stetigkeit in 1, die ja Voraussetzung für die Holomorphie ist. Aber ob das reicht?!
Mathespezialschüler Auf diesen Beitrag antworten »

Der offensichtlich knifflige Punkt ist hier einfach die Frage, ob die offene oder die abgeschlossene Einheitskreisscheibe gemeint ist. Damit die Aufgabenstellung sinnvoll wird, bin ich der Meinung, dass es die offene sein sollte.

Und dann ist das Gegenbeispiel von Dunkit korrekt.

Falls es doch die abgeschlossene Einheitskreisscheibe sein sollte, wüsste ich gern, wie ihr Holomorphie auf einer nicht offenen Menge definiert habt?!
Sly Auf diesen Beitrag antworten »

Moment mal: Was genau bedeutet jetzt die zweite Voraussetzung?

Die Aussage

bedeutet in meinen Augen:
Für alle Folgen z_n, die gegen einen Punkt auf dem Rand konvergieren, ist der betraglich größte Häufungspunkt von f(z_n) .

Oder liege ich falsch? Ich verstehe nämlich nicht, inwiefern die obigen Funktionen Gegenbeispiele sein sollen verwirrt
Mathespezialschüler Auf diesen Beitrag antworten »

Sorry, ich habe die Voraussetzung übersehen und gedacht, es würde nur gefordert, dass die Limites superior alle endlich sein sollen.

Allerdings hast du auch etwas übersehen. Das soll nur für alle gelten.

Das Beispiel von Dunkit ist natürlich kein Gegenbeispiel, da das Supremum für einige Punkte auf dem Rand trotzdem nicht bleibt.
Sly Auf diesen Beitrag antworten »

Ach ja richtig...hast recht, das habe ich wirklich total übersehen. Hammer

Hmm... aber das müsste doch mit einem Argument der Sorte "die 1 lässt sich durch die anderen Elemente auf dem Kreisrand beliebig annähern" zu bewältigen sein...wie genau, ist die Frage Big Laugh
Dunkit Auf diesen Beitrag antworten »

Hi,
Ja, die Einheitskreisscheibe ist bei uns offen.
Also ich sehe das eigtl ähnlich wie Sly: Wenn es eine solche nicht beschränkte Funktion gäbe, würde ich mir ernsthaft Sorgen um die Stetigkeit in 1 machen. Demzufolge müsste die Behauptung eigentlich wahr sein. Nur habe ich schon von einem Mitstudenten gehört, er habe ein Gegenbeispiel gefunden, und der Junge hat in der Regel recht Augenzwinkern Kann natürlich auch sein, dass er diesmal falsch liegt...
Habe gestern mal versucht, die Aussage zu beweisen, hat aber auch nichts gebracht :-/
Wenn Niemand mehr Vorschläge hat werde ich Euch zumindest am MO in einer Woche sagen können wie es denn jetzt ist, falls die Antwort nicht noch irgendwie vom Himmel fällt... sind ja noch andere Aufgaben zu erledigen Augenzwinkern
Mathespezialschüler Auf diesen Beitrag antworten »

ist doch in nicht mal definiert, von Stetigkeit kann dort also keine Rede sein. Das Problem war eben, dass nicht klar ist, ob der offene oder der abgeschlossene Ball gemeint ist. Da der offene Ball gemeint ist, erübrigen sich solche Überlegungen.

Falls es so eine Funktion gibt, lässt sie sich ja offenbar auf den Rand des Einheitskreises nicht stetig fortsetzen. Da holomorph auf ist, kann man aber in als Potenzreihe



mit Konvergenzradius Eins schreiben. Man müsste also geeignete Koeffizienten finden, um sich eine Potenzreihe definieren zu können, welche dann die gewünschte Eigenschaft haben soll. Eventuell kann man die Funktion auch ohne Potenzreihe angeben.
Dunkit Auf diesen Beitrag antworten »

Hi,
also die Koeffizienten in dieser Potenzreihe müssten doch in der Summe zumindest sein, sonst kann die Bedingung ja garnicht erfüllt sein. Wenn sie das aber sind, wieso sollte dann der Wert für plötzlich unbeschränkt sein?!
Dunkit Auf diesen Beitrag antworten »

Neuer Vorshclag für ein Gegenbeispiel (cih weiß, das geht hier etwas hin- und her Augenzwinkern )

Da kommt das nämlich jetzt meiner Meinung nach mit den Voraussetzungen hin und der GW gegen 1 ist nicht beschränkt...
Mathespezialschüler Auf diesen Beitrag antworten »

Zitat:
Original von Dunkit
also die Koeffizienten in dieser Potenzreihe müssten doch in der Summe zumindest sein

Die Koeffizienten sind eventuell echt komplexe Zahlen. Eine solche Aussage ergibt also keinen Sinn.

Warum sollten nun die Voraussetzungen erfüllt sein? Ich denke nicht, dass die Limites superior alle kleinergleich Eins sind.
Dunkit Auf diesen Beitrag antworten »

Sorry, muss natürlich betraglich sein

Hm ich denke schon: Denn die Funktion ist doch für alle beschränkt und zwar durch , wenn ich mich nicht verrechne. Wenn ich nun also durch teile, müsste ich doch die gewünschte Fkt kriegen, und da kommt genau das raus, was ich in meinem vorherigen Post genannt habe
Abakus Auf diesen Beitrag antworten »

Du müsstest vielleicht noch mal alle Eigenschaften systematisch abchecken und dann alles zusammensetzen.

Grüße Abakus smile
Mathespezialschüler Auf diesen Beitrag antworten »

Zitat:
Original von Dunkit
Denn die Funktion ist doch für alle beschränkt und zwar durch , wenn ich mich nicht verrechne.

Hm ok vielleicht stimmt das. Kannst du mal die Rechnung dazu zeigen?
Dunkit Auf diesen Beitrag antworten »

Sorry, ganzen Tag renoviert Big Laugh
Ausführliche Rechnung ist noch in Arbeit, je nachdem wann das fertig wird kann ich es noch online stellen, bevor ich das Blatt abgeben muss, sonst muss das bis Montag in einer Woche warten Augenzwinkern
Dunkit Auf diesen Beitrag antworten »

So hier nun grob meine (richtige) Lösung:

Als Gegenbeispiel wählt man die Einschränkung von auf die Einheitskreisscheibe.
Man zeigt recht leicht, dass die Funktion durch 1 beschränkt ist für alle Folgen gegen (der limes superior ist hier ein Limes und die Stetigkeit auf hilft.)
Für eine gegen 1 konvergente Folge (zB ist die Funktion aber nicht beschränkt!
Neue Frage »
Antworten »



Verwandte Themen

Die Beliebtesten »
Die Größten »
Die Neuesten »